Psy Prev 250+

Pataasin ang iyong marka sa homework at exams ngayon gamit ang Quizwiz!

Questions 256 to 262 Match the correct defense mechanism with each patient's actions. Each let- tered option may be used once, more than once, or not at all. a. Distortion b. Repression c. Reaction formation d. Sublimation e. Somatization f. Intellectualization g. Suppression h. Isolation of affect i. Introjection j. Projection k. Identification with the aggressor l. Projective identification m. Denial n. Displacement

256 to 262. The answers are 256-e, 257-g, 258-n, 259-j, 260-a, 261-k, 262-m.

Questions 263 to 265 Match each of the following vignettes with the type of psychotherapy that is being employed. Each lettered option may be used once, more than once, or not at all. a. Insight-oriented psychodynamic psychotherapy b. Supportive psychotherapy c. Short-term psychodynamic psychotherapy d. Psychoanalysis e. Cognitive therapy f. Behavioral therapy g. Experiential-humanistic therapy h. Eclectic or integrated therapy

263 to 265. The answers are 263-b, 264-f, 265-h.

286. A 30-year-old woman presents to the psychiatrist with a 2 month history of difficulty in concentrating, irritability, and depression. She has never had these symptoms before. Three months prior to her visit to the psychiatrist, the patient noted that she had experienced a short-lived flu- like illness with a rash on her calf, but has noted no other symptoms since then until the mood symptoms began. Her physical examination was within normal limits. Which of the following is the most likely diagnosis? a. Neurosyphilis b. Chronic meningitis c. Lyme disease d. Creutzfeldt-Jakob disease e. Prion disease

286-c, 287-d. (Kaplan, p 366.) Lyme disease is characterized by a bull's-eye rash at the site of the tick bite, fol- lowed by a flu-like illness which is often short-lived and may go unnoticed. Problems with cognitive functioning and mood changes may be the first complaints seen. These include problems concentrating, irritability, fatigue, and a depressed mood. Treatment consists of a 2- to 3-week course of doxycycline, which is curative about 90% of the time. If the disease is left untreated, 60% of patients will develop a chronic condition.

288. A 37-year-old woman comes to the physician with a chief complaint of a depressed mood. The patient states she has anhedonia, anergia, a 10-lb weight loss in the last 3 weeks, and states she "just doesn't care about any- thing anymore." She also admits to suicidal ideation without intent or plan. Which of the following physiologic disturbances will likely also be found in this patient? a. Decreased hypothalamic-pituitary adrenal (HPA) activity b. Increased lymphocyte proliferation in response to mitogens c. Decreased core body temperature during sleep d. Increased phasic REM sleep e. Increased anterior brain metabolism on PET scan

288-d, 289-b. (Kaplan, pp 530-531, 557.) Elevated HPA levels are a hallmark of stress in mammals, and provide a clear link between the biology of stress and depression. Lymphocytic pro- liferation in response to mitogens is decreased in depression. Both core body temperature and phasic rapid eye movement (REM) sleep are increased in those with depression. A widely replicated finding on PET scans in patients with depression is a decreased anterior brain metabolism. There are numerous other physiologic changes associated with depressions as well. The most common clinical mistake made when treating a patient with a major depression is to put the patient on a dose of an antidepressant that is too low, or is used for too short a time. Doses of antidepressants should generally be raised to their maximal doses and kept there for 4 to 5 weeks before a drug trial is considered unsuccessful. However, if a patient is doing well on a low dose of an antidepressant, that dosage should not be raised unless clinical improvement stops before the patient has reached his maximum benefit from the drug.

289. The patient in the case above is started on an SSRI. After 1 week of the medication, no improvement is seen and the dosage is raised to the maximum recommended level. For how many weeks should this new dosage be maintained before determining that the drug trial is unsuccess- ful if there is no improvement shown? a. 1to2weeks b. 4to5weeks c. 8to9weeks d. 12 to 13 weeks e. 16 or more weeks

289-b. (Kaplan, pp 530-531, 557.) Elevated HPA levels are a hallmark of stress in mammals, and provide a clear link between the biology of stress and depression. Lymphocytic pro- liferation in response to mitogens is decreased in depression. Both core body temperature and phasic rapid eye movement (REM) sleep are increased in those with depression. A widely replicated finding on PET scans in patients with depression is a decreased anterior brain metabolism. There are numerous other physiologic changes associated with depressions as well. The most common clinical mistake made when treating a patient with a major depression is to put the patient on a dose of an antidepressant that is too low, or is used for too short a time. Doses of antidepressants should generally be raised to their maximal doses and kept there for 4 to 5 weeks before a drug trial is considered unsuccessful. However, if a patient is doing well on a low dose of an antidepressant, that dosage should not be raised unless clinical improvement stops before the patient has reached his maximum benefit from the drug.

290. A 25-year-old woman delivers a healthy baby boy by Caesarean sec- tion. She notes over the next week that she has become irritable and is not sleeping very well. She worries that her child will die and fantasizes that if the child died, she would kill herself as well. She reports not being able to sleep, and has lost 10 lb within 1 week. Over the course of the following week, she begins to investigate how she might commit suicide and calls a friend to see whether the friend will babysit so that the woman will not be leaving the child alone should this occur. Which of the following is the most likely diagnosis? a. Postpartum depression b. Postpartum psychosis c. Uncomplicated bereavement d. Postpartum blues e. Generalized anxiety disorder

290. The answer is a. (Kaplan, p 865.) Postpartum blues are very fre- quent, with a prevalence estimated between 20% and 40%. Symptoms include tearfulness, irritability, anxiety, and mood lability. Symptoms usu- ally emerge during the first 2 to 4 days after birth, peak between days 5 and 7, and resolve by the end of the second week postpartum. This condition resolves spontaneously, and usually the only interventions necessary are support and reassurance. The patient in question 290 shows more severe symptomatology, including suicidal ideation with the beginnings of a plan, which may indicate postpartum depression rather than the much more fre- quent postpartum blues.

Questions 292 and 293 292. A 32-year-old woman is brought to the emergency room by the police after she was found standing in the middle of a busy highway, naked, commanding the traffic to stop. In the emergency room she is agitated and restless, with pressured speech and an affect that alternates between euphoric and irritable. Her father is contacted and states that this kind of behavior runs in the family. Which of the following is the most likely diag- nosis? a. Delirium b. Bipolar disorder, manic c. Bipolar disorder, mixed state d. Cyclothymia e. Schizophrenia

292. The answer is b. (Kaplan, pp 528, 547.) Mood elevation, mood lability, irritability, expansive behavior, increased energy, decreased need for sleep, lack of insight, poor judgment, disinhibition, impulsivity, and pressured speech are characteristic symptoms of elated acute mania. In more severe cases, mood-congruent delusional ideations and hallucinations are present.

293. The resident on call decides to start the patient above on a medication to control this disease. The patient refuses the medication, stating that she has taken it in the past and it causes her to be constantly thirsty and break out in pimples and makes her food taste funny. Which of the following medications is being discussed? a. Valproic acid b. Haloperidol c. Carbamazepine d. Lithium e. Sertraline

293. The answer is d. (Kaplan, pp 560-562.) Lithium is still the treat- ment of choice for acute mania and maintenance, although anticonvulsants such as valproate and carbamazepine have been proven effective. Newer anticonvulsants, such as gabapentin, topiramate, and lamotrigine, have also proved to have mood-stabilizing properties, although these medica- tions have not been extensively studied yet. Weight gain, metallic taste, acne, hypothyroidism, and polyuria are common complaints with long- term lithium treatment.

304. A 19-year-old woman comes to the psychiatrist for a history of anger and irritability, which occurs on monthly on an average. During this time the patient also reports feeling anxious and "about to explode," which alternates rapidly with crying spells and angry outbursts. The patient notes during this time she can't concentrate and sleeps much more than she usually needs to do. During the several days these symptoms last, the patient must skip most of her classes because she cannot function. Which of the following is the most likely diagnosis? a. Adjustment disorder with depressed mood b. Major depression c. Premenstrual dysphoric disorder d. Dysthymic disorder e. Depressive personality disorder

304. The answer is c. (Kaplan, p 867.) Premenstrual dysphoric disorder is an illness which is triggered by the changing levels of hormones which occur during a menstrual cycle. It occurs approximately 1 week before the onset of menses and is characterized by headaches, anxiety, depression, irritability, and emotional lability. Other symptoms include edema, weight gain, and breast pain. Approximately 5% of women suffer from this disor- der. Some patients respond to short courses of SSRIs, in addition to symp- tomatic treatment with analgesics and diuretics.

310. A 45-year-old woman comes to her physician for help with her insomnia. She states "ever since my husband died, I just can't sleep." The patient states her 57-year-old husband died suddenly of a heart attack 9 weeks ago. Since that time, the patient has had a very depressed mood, had been crying, has lost interest in activities, is fatigued, and has insomnia. Which of the following symptoms, if present, should make the physician think this patient has a major depression instead of bereavement? a. The patient feels that she would be better off dead. b. The patient has marked functional impairment. c. The patient has lots of guilt about not recognizing that the chest pain her hus- band was having was the start of a heart attack. d. The patient has mild psychomotor retardation. e. The patient reports hearing the voice of her dead husband calling her name twice.

310. The answer is b. (Kaplan, p 889.) This patient is suffering from bereavement, which normally begins immediately after, or within a short time of the death of a loved one. There are certain symptoms which are not characteristic of a "normal" grief reaction and may help in the differentia- tion of bereavement from a major depression. These include (1) guilt about things other than actions taken or not taken by the survivor at the time of the loved one's death, (2) thoughts of death other than the survivor feeling he/she would be better off dead without the loved one, (3) a morbid pre- occupation with worthlessness, (4) marked psychomotor retardation, (5) marked and prolonged functional impairment, and (6) hallucinations other than the survivor believing he/she can hear the voice or see the loved one.

333. A 26-year-old woman comes to the psychiatrist with a 1-month his- tory of severe anxiety. The patient states that 1 month ago she was a "nor- mal, laid-back person." Since that time she rates her anxiety an 8 on a scale of 1 to 10, and also notes she is afraid to leave the house unless she checks that the door is locked at least five times. Which of the following medical conditions could commonly cause this kind of symptom presentation? a. Hyperglycemia b. Crohn's disease c. Hyperparathyroidism d. Fibromyalgia e. Peptic ulcer disease

333. The answer is c. (Kaplan, p 354.) Medical conditions which can cause anxiety-related symptoms include the endocrinopathies (pheochro- mocytoma, hyperthyroidism, hypercortisolemic states, and hyperparathy- roidism), metabolic problems (hypoxemia, hypercalcemia, and hypoglycemia) and neurologic disorders, including vascular, trauma, and degenerative types. Although mitral valve prolapse and panic attacks have long been associated, the mitral valve prolapse actually causing the panic attacks is not known.

353. An attractive and well-dressed 22-year-old woman is arrested for prostitution, but on being booked at the jail, she is found to actually be a male. The patient tells the consulting physician that he is a female trapped in a male body and he has felt that way since he was a child. He has been taking female hormones and is attempting to find a surgeon who would remove his male genitals and create a vagina. Which of the following is the most likely diagnosis? a. Homosexuality b. Gender identity disorder c. Transvestic fetishism d. Delusional disorder e. Schizophrenia

353. The answer is b. (Moore and Jefferson, pp 191-205.) In adolescents and young adults, gender identity disorder is characterized by strong cross- gender identification, a persistent discomfort with one's sex, and clinically significant distress or impairment. Such patients usually trace their conviction to early childhood, often live as the opposite sex, and seek sex reas- signment surgery and endocrine treatment. These patients feel a sense of relief and appropriateness when they are wearing opposite-sex clothing. In contrast, patients with transvestic fetishism are sexually aroused by this behavior, and so typically only seek to wear clothing of the opposite sex during sexual situations. Homosexuality is not a diagnosis in DSM-IV. While some homosexuals cross-dress to seek a same-sex partner, they do not feel that they belong to the opposite sex, nor do they seek sex reassign- ment surgery.

Questions 370 to 371 370. A young librarian has been exceedingly shy and fearful of people since childhood. She longs to make friends, but even casual social interac- tions cause her a great deal of shame and anxiety. She has never been at a party, and she has requested to work in the least active section of her library, even though this means lower pay. She cannot look at her rare cus- tomers without blushing, and she is convinced that they see her as incom- petent and clumsy. Which of the following personality disorders is most likely? a. Schizotypal b. Avoidant c. Dependent d. Schizoid e. Paranoid 371. Which anxiety disorder is most likely to be confused with the per- sonality disorder described in the vignette above? a. Generalized anxiety disorder b. Specific phobia c. Agoraphobia d. Social phobia e. Obsessive-compulsive disorder

370: The answer is b. (Jacobson, p 187.) Avoidant personality disorder is characterized by pervasive and excessive hypersensitivity to negative eval- uation, social inhibition, and feelings of inadequacy. Impairment can be severe because of social and occupational difficulties. Males and females are equally affected. The prevalence ranges from 0.5% to 1.5% in the general population. Among psychiatric outpatients, the prevalence is as high as 10%. Patients with avoidant personality disorder would like friends, but are so afraid that they will be rejected that they do not try to make them. Patients with schizoid personality disorders, by contrast, are socially iso- lated and prefer it that way. 371: he answer is d. (Jacobson, p 187.) Avoidant personality disorder can be difficult to differentiate from social phobia. One differentiating factor is that in social phobia, specific situations rather than interpersonal contact cause distress and are avoided.

Match each patient's behavior with the most likely personality disorder. Each lettered option may be used once, more than once, or not at all. a. Paranoid b. Schizotypal c. Schizoid d. Narcissistic e. Borderline f. Histrionic g. Antisocial h. Obsessive-compulsive i. Dependent j. Avoidant k. No personality disorder apparent 384. A 24-year-old woman drops out of college after 2 weeks. When asked why, she states that although she would desperately like to have friends, she is afraid to approach anyone because "they would think I'm just a nerd." Furthermore, in the middle of a class, one of the professors asked her a question and she became extremely uncomfortable. She has never had a significant relationship with anyone other than her parents and sister. 385. A 32-year-old man comes to the psychiatrist because he is anxious about his new job. He notes that he previously held a job shelving books in the back of a library, but because of budget cuts he has been forced to inter- act with customers. He states he doesn't like being around people and prefers being by himself. He appears emotionally cold and detached during the interview. 386. A 19-year-old man comes to the psychiatrist because he can't leave the house without checking the stove, furnace, and water heater 25 times in a specific order. He notes that while he hates to perform this behavior, if he does not, he feels overwhelmingly anxious. It sometimes takes him 3 hours to leave the house in the morning because of this behavior. 387. A 32-year-old woman is admitted to the obstetrics ward to deliver a normal full-term infant. Ten hours after the delivery, she tries to steal the infant out of the nursery because she believes that the government of Myanmar is after her and will steal her child. When confronted by a nurse, she attempts to scratch the nurse and grab her child.

384-j, 385-c, 386-k, 387-k. Avoidant personality disorder is characterized by an 224 Psychiatry intense need for connection and social interaction with others, coupled with an intense fear of rejection. This fear causes patients to avoid any new or social situations that might be potentially embarrassing, and to feel extremely inadequate about the ability to start and maintain any kind of relationship. Conversely, patients with schizoid personality disorder, while every bit as isolated as those with avoidant personality disorder, like it that way. They rarely come to psychiatric attention because their social isolation is ego-syntonic. They do not like social relationships and usually prefer iso- lated activities. They have few, if any, friends. Question 386 refers to a patient with obsessive-compulsive disorder (not obsessive-compulsive per- sonality disorder). Patients with OCD are characterized by obsessions (in this case, that there is something wrong with the stove, furnace, and water heater) and compulsions (the checking activity). These rituals can very much disturb a patient's life and are almost always ego-dystonic

Questions 391 and 392 391. A 50-year-old man is brought to the emergency department by ambulance. His respirations are shallow and infrequent, his pupils are con- stricted, and he is stuporous. He was noted to have suffered a grand mal seizure in the ambulance. Which of the following drugs is this man most likely to have overdosed on? a. Cocaine b. LSD c. Meperidine d. PCP e. MDMA (Ecstasy) 392. After ensuring adequate ventilation for the patient in the previous vignette, which of the following interventions should be next? a. Intravenous naloxone b. Intravenous phenobarbitol c. Intravenous diazepam d. Forced diuresis e. Intramuscular haloperidol

391. The answer is c. (Jacobson, pp 105-106.) Severe opiate intoxication is associated with respiratory depression, stupor or coma, and sometimes pulmonary edema. Less severe intoxication is associated with slurred speech, drowsiness, and impaired memory or attention. Early on, the pupils are constricted, but they dilate if the patient becomes anoxic because of the respiratory depression. Blood pressure is typically reduced. Meperidine intoxication in a chronic user is often complicated by delirium or seizures caused by the accumulation of normeperidine, a toxic metabo- lite with cerebral irritant properties. 392. The answer is a.

388. A 19-year-old man is brought to the emergency room by his dis- traught parents, who are worried about his vomiting and profuse diarrhea. On arrival, his pupils are dilated, his blood pressure is 175/105 mm Hg, and his muscles are twitching. His parents report that these symptoms started 2 hours earlier. For the past few days he has been homebound because of a sprained ankle, and during this time he has been increasingly anxious and restless. He has been yawning incessantly and has had a runny nose. Which of the following drugs is this man most likely to be with- drawing from? a. Heroin b. Alcohol c. PCP d. Benzodiazepine e. Cocaine 389. The physician tells this patient from the vignette above that the with- drawal symptoms will peak how long after stopping the use of this sub- stance? a. 6 hours b. 15 hours c. 48 hours d. 3 days e. 1 week

A & C

287. Which of the following medications should be used to treat the patient above? a. Penicillin b. Antiviral medication c. Amphotericin B d. Doxycycline e. Prozac for depressed mood (ie, treat the depressed mood only)

D

332. A 23-year-old woman presents to her physician with the chief com- plaint that she is anxious about the way she looks. She notes that for "as long as she can remember," she has been obsessed about the fact that some- thing must be wrong with her face. She notes that her eyes are too far apart and her nose is misshapen. She states that this concern is "ruining her life" because she spends all her time isolated from others so that they cannot see her face. The physician did not notice anything unusual about the patient's face, but the patient cannot be consoled by this statement. Which of the following is the most likely diagnosis? a. Body dysmorphic disorder b. Delusional disorder c. Obsessive-compulsive disorder d. Somatization disorder e. Hypochondriasis

The answer is a. (Kaplan, p 635.) Body dysmorphic disorder is characterized by a preoccupation with an imagined defect in appearance (in this case, the patient believes that her eyes and nose are malformed). The preoccupation must interfere with normal daily functioning and cause clinically significant distress. It cannot be better accounted for by another mental disorder (such as anorexia nervosa). This disorder has an approxi- mately equal distribution between men and women, and most patients are 20 to 40 years old at time of diagnosis.

374. A 21-year-old man comes to the psychiatrist with the complaint of chronic unhappiness. He states that his usual mood is unhappy, and his self- esteem is always low. He states he spends time brooding and worrying about all manner of issues, in particular over his own inadequacy. He states he is pessimistic about things as a general rule, and feels guilty a lot because he is "such a bad friend." Which of the following is the most likely diagnosis? a. Personality disorder NOS b. Dysthymic disorder c. Major depression d. Avoidant personality disorder e. Generalized anxiety disorder

The answer is a. (Kaplan, p 808.) This patient does not meet the criteria for any one current personality disorder, and thus would receive a diagnosis of Personality Disorder NOS. Depressive personality disorder, part of DSM-IV-TR's research criteria, would actually fit this patient's symp- tomatology well. In this disorder, there is a pervasive pattern of depressive cognition and behavior which begins in early adulthood and persists throughout life. Patients with this personality disorder are pessimistic, anhedonic, self-doubting, and chronically unhappy. Their moods do not fluctuate as much as those with dysthymic disorder, as dysthymia is episodic and usually occurs after some precipitating event. These patients may be at a great risk for developing dysthymia or a major depression.

315. A middle-aged woman presents with a variety of cognitive and somatic symptoms, fatigue, and memory loss. She denies feeling sad, but her family physician is aware of this patient's lifelong inability to identify and express feelings. He suspects she is depressed. Which of the following results is most likely to confirm a diagnosis of depression? a. Reduced metabolic activity and blood flow in both frontal lobes on PET scan b. Diffuse cortical atrophy on CAT scan c. Atrophy of the caudate on MRI d. Prolonged REM sleep latency in a sleep study e. Subcortical infarcts on MRI

The answer is a. (Kaplan, pp 81-85.) Positron emission tomogra- phy (PET) scan has consistently demonstrated a decrease in blood flow and metabolism in the frontal lobe of depressed patients. Most studies have found bilateral rather than unilateral deficits and equivalent decreases in several types of depression (unipolar, bipolar, associated with OCD). Cor- tical atrophy and subcortical infarcts are associated, respectively, with Alzheimer disease and multi-infarct dementia. Atrophy of the caudate is characteristic of Huntington's disease. In major depression, the REM sleep latency (the period of time between falling asleep and the first period of REM sleep) is shortened, not prolonged.

394. A college freshman, who has never consumed more than one occa- sional beer, is challenged to drink a large quantity of alcohol during his fra- ternity house's party. In a nontolerant person, signs of intoxication usually appear when the blood alcohol level reaches what range? a. 20 to 30 mg/dL b. 100 to 200 mg/dL c. 300 mg/dL d. 400 mg/dL e. 500 mg/dL

The answer is a. (Moore and Jefferson, pp 89-90.) Behavioral changes, slowing of motor performance, and decrease in the ability to think clearly may appear with a blood alcohol level as low as 20 to 30 mg/dL. Most people show significant impairment of motor and mental perfor- mance when their alcohol levels reach 100 mg/dL. With blood alcohol con- centration between 200 and 300 mg/dL, slurred speech is more intense and memory impairment, such as blackout and anterograde amnesia, becomes common. In a nontolerant person, a blood alcohol level over 400 mg/dL can produce respiratory failure, coma, and death. Because of toler- ance, chronic heavy drinkers can present with fewer symptoms, even with blood alcohol levels greater than 500 mg/dL.

328. A 28-year-old taxi driver is chronically consumed by fears of having accidentally run over a pedestrian. Although he tries to convince himself that his worries are silly, his anxiety continues to mount until he drives back to the scene of the "accident" and proves to himself that nobody lies hurt in the street. This behavior best exemplifies which of the following? a. A compulsion secondary to an obsession b. An obsession triggered by a compulsion c. A delusional ideation d. A typical manifestation of obsessive-compulsive personality disorder e. A phobia

The answer is a. (Jacobson, pp 85-91.) Recurrent obsessions and compulsions are essential features of obsessive-compulsive disorder (OCD). Obsessions are persistent thoughts or mental images that are sub- jectively experienced as intrusive and alien and characteristically provoke various levels of anxiety. Compulsions are repetitive acts, behaviors, or thoughts designed to counteract the anxiety elicited by the obsessions. Thus obsessions (which cause anxiety) are paired with their related com- pulsions (which help manage the anxiety produced). The diagnosis of obsessive-compulsive personality disorder is reserved for those patients with significant impairments in their occupational or social effectiveness. These patients are preoccupied with rules, regulations, orderliness, neat- ness, details, and the achievement of perfection.

393. A 22-year-old man arrives at an emergency room accompanied by several friends. He is agitated, confused, and apparently responding to frightening visual and auditory hallucinations. The patient is put in restraints after he tries to attack the emergency room physician. The patient's friends report that he had "dropped some acid" 6 or 7 hours earlier. How much longer will intoxication with this substance last? a. 1to6hours b. 8 to 12 hours c. 14 to 18 hours d. 20 to 24 hours e. 26 to 30 hours

The answer is a. (Jacobson, pp 115-117.) Most cases of intoxication with a hallucinogen are over within 8 to 12 hours, but prolonged drug- induced psychoses may occur, especially with phencyclidine (PCP), from which the psychosis may last several weeks.

334. A 33-year-old man comes to his physician for an HIV test. The test is positive. The patient has no signs or symptoms of AIDS. Which of the fol- lowing psychiatric diagnoses may develop in as many as 25% of patients informed of a positive HIV test? a. Adjustment disorder with anxiety b. PTSD c. Bipolar disorder, manic d. Panic disorder e. Hypochondriasis

The answer is a. (Kaplan, p 375.) Patients receiving a positive HIV test may need counseling for a wide variety of symptoms, including anxi- ety and depression, a syndrome like PTSD, concern about minor physical symptoms, and adjustment disorders, the latter of which can occur in as many as 25% of those given a positive diagnosis. Other issues may be wor- ries about telling friends and family, issues of guilt and self-esteem, worries about financial and work considerations, and the future of the medical condition itself.

356. A 65-year-old retired steelworker who has never had any sexual dys- function experiences difficulty in obtaining and maintaining an erection shortly after he starts taking a medication prescribed by his primary care physician. Which of the following medications is most likely to cause such a side effect? a. Propranolol b. Amoxicillin c. Lorazepam d. Bupropion e. Thyroid hormones

The answer is a. (Kaplan, p 696.) Among beta-blockers, propra- nolol is the most likely to cause impotence. Furthermore, through its effect on the serotonin system, propranolol can also inhibit orgasm and reduce sex drive. Fatigue and depressed mood--also frequent side effects of pro- pranolol--can likewise have a negative effect on sexual function.

314. A 32-year-old man is admitted to the psychiatric unit after his wife brought him to the emergency room in a severe major depression. The patient signs himself in voluntarily because he "didn't think he is safe" at home. Which of the following factors most increases a patient's risk of sui- cide while on the inpatient unit? a. The patient is in his first week of hospitalization. b. Staff morale is high on the unit. c. The patient is admitted in early July (new residents are on the unit). d. The patient is started on an SSRI the first day on the unit. e. The patient is told he will be evaluated for ECT.

The answer is a. (Kaplan, p 899.) For both men and women, the risk of suicide is highest in the first week of hospitalization. After 3 to 5 weeks as an inpatient, suicide risks drops to the same level as that of the general population. Times of staff rotation, such as the end of June when the old residents are rotating off the unit, and times of staff demoralization, are also periods where higher suicidal risk is found.

312. A 35-year-old woman is seeing a psychiatrist for treatment of her major depression. After 4 weeks on fluoxetine at 40 mg/day, her psychia- trist decides to try augmentation. Which of the following is the most appropriate medication? a. Lithium b. Sertraline c. An MAO inhibitor d. Clonazepam e. Haloperidol

The answer is a. (Kaplan, pp 1056-1058.) Lithium has been proven effective when added to an antidepressant in the treatment of refractory depression. More than one mechanism of action is probably involved, although lithium's ability to increase the presynaptic release of serotonin is the best understood. Other augmentation strategies include the use of thyroid hormones, stimulants, estrogens, and light therapy. The combination of two SSRIs (in this case, fluoxetine and sertraline) or of an MAOI and an SSRI is not recommended because of the risk of precipitating a serotonin syndrome.

313. Which of the following is a relative contraindication for ECT? a. Space-occupying lesion in the brain b. Pregnancy c. Hypertension d. Seizure disorder e. Status post-myocardial infarction 6 months earlier

The answer is a. (Kaplan, pp 1117-1120.) ECT is a safe procedure with very few contraindications (recent myocardial infarcts, increased intracranial pressure, aneurysms, bleeding disorders, and any condition that disrupts the blood-brain barrier).

270. The parents of a 20-year-old schizophrenic are having difficulty deal- ing with their son's decline in function. Once a good student with friends and a social life, the son now spends his days barricaded in his room, mum- bling to himself, or watching the street with binoculars. Which of the fol- lowing family interventions would be most helpful in this situation? a. Teaching the parents about reducing expressed emotions in the family's interactions b. Unmasking the family game and freeing the identified patient from the role of symptom bearer c. Encouraging the parents to openly discuss their feelings of loss and disappoint- ment with their son d. Discussing the secondary gains provided by the son's symptoms e. Discussing the parents' marital problems and how the son's disorder affects them

The answer is a. (Kaplan, pp 940-942.) Family interventions that have been shown to be effective in the treatment of schizophrenic patients include teaching the family members about schizophrenia, emphasizing the importance of keeping interpersonal communication at a low emotional quotient (schizophrenic patients tend to relapse when exposed to the intense negative emotions of family members), and helping the family learn more adaptive ways to cope with stress. Discussing marital problems in front of the patient and sharing with the patient the distressing details of the parents' own struggles with his or her mental illness are bound to have neg- ative effects. Uncovering the family game was one of the goals of systemic family therapy created by Selvini-Palazzoli and the Milan group. This model was accepted in the 1960s, when schizophrenia was considered the conse- quence of pathological parenting. In view of what is now known about schizophrenia's biological etiology, this theory is no longer considered valid.

351. A 17-year-old man comes to the physician because he has been falling asleep in inappropriate places, even though he has been getting enough rest at night. The patient states that he has fallen asleep while eat- ing and driving. He notes that he stays asleep approximately 20 minutes and when he first wakes up, he is unable to move. He notes that sometimes he can even fall asleep while standing, and has been told by others that during those times he simply drops to the floor suddenly. He is fitted with a portable monitor, and it is found that during these episodes he enters an REM sleep stage immediately. Which of the following is the most likely diagnosis? a. Narcolepsy b. Sleep apnea c. Primary hypersomnia d. Kleine-Levin syndrome e. REM sleep behavior disorder

The answer is a. (Moore and Jefferson, pp 219-221.) In narcolepsy, REM periods are not segregated in their usual rhythm during sleep but suddenly and repeatedly intrude into wakefulness. Nocturnal sleep shows a sleep-onset REM period or one that occurs very shortly after the onset of sleep. Among patients treated for this disorder, 15% to 30% also show some nocturnal myoclonus or sleep apnea. A majority of narcoleptics expe- rience cataplexy (a sudden loss of muscle tone), hypnagogic hallucinations (dreamlike experiences occurring just before real sleep occurs), or sleep paralysis (brief paralysis occurring just before, or just after, the onset of sleep).

294. A 28-year-old woman is diagnosed with bipolar disorder, manic type, when she was hospitalized after becoming psychotic, hypersexual, severely agitated, and unable to sleep. She is started on a medication in the acute phase of her illness. Which of the following medications, recom- mended for acute use in manic patients, is recommended to be continued on into maintenance therapy? a. Aripiprazole b. Lamotrigine c. Lithium d. Olanzepine e. Ziprasidone

The answer is c. (Kaplan, p 561.) Lithium, carbamazepine, and val- proic acid are the agents most often used in the long-term maintenance treatment of bipolar disorder. Antipsychotics and benzodiazepines, while very helpful in the acute treatment of bipolar mania, should be discontinued as soon as the patient is in a stable state and not used in long-term mainte- nance unless absolutely necessary, given the risk of movement disorders and dependence, respectively.

349. A 7-year-old girl is brought to the physician because her parents note that she gets up at night and, still asleep, walks around the house for a few min- utes before returning to bed. When she is forced to awaken during one of these episodes, she is confused and disoriented. Her parents are afraid that she will accidentally hurt herself during one of these episodes. Which of the following is the most appropriate intervention the physician should recommend? a. Tell the parents to maintain a safe environment and monitor the patient's symptoms b. Start the patient on a low dose of benzodiazepines at night c. Start the patient on a low dose of a tricyclic antidepressant d. Tell the parents that the child would benefit from cognitive psychotherapy e. Admit the child to the hospital and obtain an EEG

The answer is a. (Moore and Jefferson, pp 234-235.) Sleepwalking disorder is a parasomnia associated with slow-wave sleep. The patient is usu- ally difficult to awaken, confused, and amnesic for the episode. Common in children, sleepwalking peaks between the ages of 4 and 8 years and usually disappears after adolescence. The person attempting to awaken the sleep- walker may be violently attacked.

307. A 64-year-old man is admitted to the psychiatric unit after an unsuc- cessful suicide attempt. Following admission, he attempts to cut his wrists three times in the next 24 hours and refuses to eat or drink anything. He is scheduled to have electroconvulsive therapy (ECT) because he is so severely depressed that an antidepressant is deemed too slow acting. Which of the following side effects should the patient be informed is most common after ECT? a. Headache b. Palpitations c. Deep venous thromboses d. Interictal confusion e. Worsening of the suicidal ideation

The answer is a. (Moore and Jefferson, pp 518-521.) The most com- mon complaints after ECT include headaches, nausea, and muscle sore- ness. Memory impairment (both retrograde and anterograde) does occur but less frequently, and interictal confusion is quite uncommon. Likewise, cardiovascular changes do occur, but they are rare and happen mostly in the immediate postictal period or during the seizure itself.

354. A 38-year-old man comes to his physician with complaints of impaired ejaculation. He is on the following medications: perphenazine, digoxin, and propranolol. He is also receiving methadone treatment and admits to periodic cannabis use. Which substance is the most likely culprit in his problems with ejaculation? a. Perphenazine b. Digoxin c. Propanolol d. Methadone e. Cannabis

The answer is a. (Kaplan, p 696.) Perphenazine is known to cause impairment in ejaculation. The other drugs in the option list can cause impaired erections, but do not generally cause problems with ejaculation once an erection is achieved.

254. A 45-year-old woman comes to a therapist with the chief complaint of feeling depressed. The therapist asks the patient to talk about her expe- riences, both in daily life and in the past. As the therapy progresses, the patient realizes that much of her depressive emotion comes from her feel- ings of abandonment as a child, when her mother was hospitalized for a long illness and was thus unavailable. The patient sees the therapist once a week. The therapist uses primarily clarification, confrontation, and inter- pretation as tools. Which of the following therapies is this patient most likely undergoing? a. Dynamic psychotherapy b. Cognitive therapy c. Behavioral therapy d. Psychoanalysis e. Experiential-humanistic psychotherapy

The answer is a. (Ebert, pp 153-155.) This patient is undergoing dynamic psychotherapy. Psychoanalysis would also use the understanding of the patient and the recreation of the past through clarification, con- frontation, and interpretation, but typically the patient comes to the office more frequently than once per week, and often, though not always, the patient lies on a couch facing away from the therapist. A cognitive therapy would most likely focus on the negative worldview of this patient and attempt to restructure those thoughts. A behavioral therapist would most likely instruct the patient to change his/her behavior as an antecedent to recovery (such as exercising or another activity). An experiential-humanis- tic therapist would focus on developing a supportive and gratifying rela- tionship with the patient to help provide the empathic responsiveness that was hypothesized as absent in the patient's past.

269. A 29-year-old woman is in psychodynamic psychotherapy for a long- standing inability to have close and meaningful relationships. During her sessions with the therapist, she often comes 10 minutes late or misses ses- sions altogether. At the beginning of the next session after a session has been missed, the therapist points out this behavior to the patient. These comments best represent which kind of therapeutic intervention? a. Confrontation b. Interpretation c. Clarification d. Desensitization e. Flooding

The answer is a. (Kaplan, pp 926-928.) As noted in question 268, confrontation as a therapeutic technique points out a particular behavior of the patient and tries to assign some meaning to it--typically that the patient is behaving in this manner in an attempt to avoid something. In this question, the patient is arriving late and missing appointments; the thera- pist, via confrontation, points out the behavior, then may note that such behavior is designed to remove the patient from therapy, perhaps because she is anxious, angry, or ambivalent about the therapy itself.

359. A 36-year-old woman comes to her physician for help with weight reduction. Her BMI is 32. Which of the following disorders is she most at risk for at this weight? a. Leukemia b. Breast cancer c. Colon cancer d. Pseudogout e. Rheumatoid arthritis

The answer is b. (Kaplan, p 745.) There are numerous health prob- lems thought to be caused or exacerbated by obesity. In women, there is an increased risk of cancer of the endometrium, breast, cervix, ovary, and gall- bladder. Osteoarthritis of the knees and spine is a concern, as is gout (not pseudogout).

357. A 38-year-old married man comes to the psychiatrist because he felt his "sexuality is out of control." He notes that he never feels that he has had enough sex, even though he masturbates 3 to 4 times per day and has sex with his wife daily. He states he has tried to stop but feels he can not con- trol the behavior. He feels a lot of guilt about this, especially when he mas- turbates at his workplace. Which of the following medications would be most helpful to this man? a. Benzodiazepines b. SSRIs c. Antipsychotics d. Mood stabilizers e. Buspirone

The answer is b. (Kaplan, pp 715-716.) Because SSRIs can reduce the sex drive, these drugs can be used to treat sexual addiction. In this case, the drug's side effects can be used therapeutically. Medroxyprogesterone acetate also diminishes libido in men, so may also be effective in the treat- ment of sexually addictive behavior.

322. A middle-aged man is chronically preoccupied with his health. For many years he feared that his irregular bowel functions meant he had can- cer. Now he is very worried about having a serious heart disease, despite his physician's assurance that the occasional "extra beats" he detects when he checks his pulse are completely benign. Which of the following is the most likely diagnosis? a. Somatization disorder b. Hypochondriasis c. Delusional disorder d. Pain disorder e. Conversion disorder

The answer is b. (Moore and Jefferson, pp 181-183.) Hypochondri- asis is characterized by fear of developing or having a serious disease, based on the patient's distorted interpretation of normal physical sensations or signs. The patient continues to worry even though physical examinations and diagnostic tests fail to reveal any pathological process. The fears do not have the absolute certainty of delusions. Hypochondriasis can develop in every age group, but onset is most common between 20 and 30 years of age. Both genders are equally represented, and there are no differences in prevalence based on social, educational, or marital status. The disorder tends to have a chronic, relapsing course.

297. What percentage of new mothers is believed to develop postpartum depression? a. <1% b. 10% to 15% c. 25% to 30% d. 35% to 40% e. >50%

The answer is b. (Kaplan, p 865.) Postpartum depression is rela- tively common, occurring in about 10% to 15% of new mothers. Symp- toms are indistinguishable from those characteristic of nonpsychotic major depression and usually develop insidiously over the 6 months following delivery. Some women, however, experience an acute onset of symptoms immediately after delivery, and occasionally depression starts during preg- nancy. Ambivalence toward the child and doubts about the mother's own parenting abilities are common, but the rate of suicide is low.

296. A 26-year-old man comes to the physician with the chief complaint of a depressed mood for the past 5 weeks. He has been feeling down, with decreased concentration, energy, and interest in his usual hobbies. Six weeks prior to this office visit, he had been to the emergency room for an acute asthma attack and was started on prednisone. Which of the following is the most likely diagnosis? a. Mood disorder secondary to a general medical condition b. Substance-induced mood disorder c. Major depression d. Adjustment disorder e. Dysthymia

The answer is b. (Kaplan, pp 573-577.) According to DSM-IV cri- teria, patients developing a mood disorder after using a substance (either illicit or prescribed) are diagnosed with a substance-induced mood disor- der. The diagnosis of major depression cannot be made in the presence of either substance use or a general medical condition that might be the cause of the mood disorder. Prednisone is a common culprit in causing mood disorders ranging from depression to mania to psychosis.

308. A 14-year-old boy is brought to the psychiatrist because for the past 15 months he has been irritable and depressed almost constantly. The boy notes that he has difficulty concentrating, and he has lost 5 lb during that time period without trying. He states that he feels as if he has always been depressed, and he feels hopeless about ever feeling better. He denies suici- dal ideation or hallucinations. He is sleeping well and doing well in school, though his teachers have noticed that he does not seem to be able to con- centrate as well as he had previously. Which of the following is the most likely diagnosis? a. Major depression b. Dysthymic disorder c. Mood disorder secondary to a general medical condition d. Normal adolescence e. Cyclothymia

The answer is b. (Moore and Jefferson, pp 142-143.) This patient is suffering from a dysthymic disorder, characterized by an irritable or depressed mood for at least 1 year. (This patient is an adolescent--if he were an adult, the time requirement for the diagnosis of dysthymic would be 2 years.) The patient complains of difficulty concentrating and has had some weight loss. He also feels hopeless about ever not feeling depressed. However, he has no suicidal ideation or psychotic symptoms and no other vegetative symptoms. He is still doing well in school, a clue that the depres- sion is probably not severe enough to rate a diagnosis of major depression, especially when combined with the length of time that this patient has been depressed and irritable.

252. A 36-year-old woman comes to see a psychiatrist with the chief com- plaint, "I feel like I'm going to die, I get so anxious." She describes occa- sions when she feels her heart beat faster, and then suddenly she becomes overwhelmed with the notion that she is going to die or go crazy. She notes that she hyperventilates, her fingers and toes tingle, and she feels as if her heart is going to break because it beats so fast. Which of the following should the psychiatrist, a cognitive therapist, urge the patient to do during their visit? a. Recognize that a past history of trauma probably contributes to these symptoms b. Recreate the panic attack c. Talk through the nature of the emotional supports that the patient has in her life d. Take medication to suppress the panic attacks, thus lessening her anxiety e. Free-associate feelings and thoughts about the panic attacks themselves

The answer is b. (Kaplan, p 596.) Patients with panic disorder often misinterpret otherwise innocuous bodily sensations (in this case, the patient's sensation of her heart beating) as signs of impending physical doom. The therapist encourages the patient to recreate the panic attack in the office to demonstrate that, though they are uncomfortable, the attacks themselves will not cause the patient to die, have a heart attack, or go crazy. In essence, this kind of therapy encourages patients to face their fears and see that they are experiencing an irrational or unrealistic concern with them. Once the patient in this question experiences the panic attack in the office and nothing terrible occurs, she will begin to prove to herself that her initial bodily sensations were no cause for alarm.

267. A 42-year-old woman comes to the psychiatrist asking for help with recovering a memory from her past. She was in a bad car accident and had become obsessed with knowing whether or not she had seen before impact the car that had hit her. She asked to be treated with hypnosis to recover this memory. Which of the following are relatively contraindicated in the use of hypnosis? a. A history of panic attacks b. The presence of paranoid delusions c. Avoidant personality disorder d. Pregnancy e. Major depression

The answer is b. (Kaplan, p 964.) Hypnosis is not recommended for suspicious and paranoid patients, who are likely to respond negatively to the loss of control that hypnosis evokes. These patients usually refuse to cooperate with the hypnotic inductions. Another category of patients who may have unplanned and potentially negative reactions to hypnosis are individuals with a history of trauma, who may undergo spontaneous abreactions.

268. A patient in psychotherapy is always anxious to please. Recently, he stated that he has begun to feel frightened in the presence of the therapist and that he has had fantasies about the analyst attacking him. Subse- quently, the patient talks about his father and his lifelong struggle to please him at any cost. After listening to these comments, the therapist says that the patient's fantasies about him appear to be closely connected with the patient's way of relating to his father. The therapist also says that the pas- sive and compliant relationship the patient has with his idealized father may represent a reaction to his fear of his father's retaliation. These com- ments best represent which kind of therapeutic intervention? a. Confrontation b. Interpretation c. Clarification d. Desensitization e. Flooding

The answer is b. (Kaplan, pp 926-928.) Interpretations, the corner- stone of psychoanalytic psychotherapy, are explanatory statements made by the analyst that link a symptom, a behavior, or a feeling to its uncon- scious meaning. Ideally, interpretations help the patient become more aware of unconscious material that has come close to the surface. Con- frontation and clarification are also used in psychoanalytic psychotherapy. In confrontation, the analyst points out to the patient something that the patient is trying to avoid. Clarification refers to putting together the infor- mation the patient has provided so far and reflecting it back to him or her in a more organized and succinct form. Flooding and desensitization are exposure techniques used in behavioral therapy.

309. A 29-year-old man is brought to the hospital because he was found running around on the streets with no shoes on in the middle of winter, screaming to everyone that he was going to be elected president. Upon admission to the hospital, he was stabilized on olanzapine and lithium and then discharged home. Assuming the patient is maintained on the olanza- pine and the lithium, which of the following tests should be performed at least once per year? a. MRI of the brain b. Liver function tests c. Creatinine level d. Rectal exam to look for the presence of blood in the stool e. ECG

The answer is c. (Jacobson, p 263.) This patient, likely suffering from bipolar disorder, mania, is being maintained on lithium and an antipsychotic. Patients on lithium, at minimum, should be monitored for the following: plasma lithium level (once every month or two until the patient is stable, and then less frequently if he or she is reliable), thyroid function tests, creatinine, and urinalysis. ECGs are part of the list of optional recommendations for patients on lithium but generally are reserved for patients over the age of 50. There are no standard blood tests or other examinations to monitor use of olanzapine.

348. A 47-year-old man with a master's degree in chemistry lives alone in a halfway house and subsists on panhandling and collecting redeemable cans. Ten years ago he lost his job in a large firm because he was found to have repeatedly stolen company money and used it to bet on horse racing. Afterward, he had several other jobs but always lost them because he stole money. He also stole and borrowed money from friends and relatives. When asked about this behavior, the patient stated that he felt very guilty about it but "couldn't seem to stop" himself. Which of the following diag- noses best fits this patient's symptoms? a. Antisocial personality disorder b. Conduct disorder c. Pathological gambling d. Fugue state e. Kleptomania

The answer is c. (Jacobson, pp 148-153.) Pathological gambling is found in the diagnostic category called impulse control disorders, together with pyromania, intermittent explosive disorder, trichotillomania, klepto- mania, and impulse control disorder not otherwise specified. The DSM-IV criteria for pathological gambling are quite similar to the criteria for sub- stance abuse disorders: preoccupation with gambling; need to gamble increasing amounts of money to obtain the same effects; unsuccessful efforts to stop or cut back; using gambling to escape problems or relieve dysphoric mood; after money losses, returning to gambling the next day to "break even"; lying to family and friends to conceal losses and the extent of the involvement in gambling; committing illegal acts to finance the habit; and jeopardizing important relationships or losing jobs because of gam- bling. The vast majority of compulsive gamblers are men. The prevalence is estimated to be 0.2% to 3% of the general population, varying with the number of gaming venues available. The diagnosis in this case is not an antisocial personality disorder because the patient feels guilty about these actions, and not kleptomania because the stealing is not the primary symp- tom (the gambling is, which has led to a need to steal to support himself).

350. A 65-year-old woman lives alone in a dilapidated house, although her family members have tried in vain to move her to a better dwelling. She wears odd and out-of-fashion clothes and rummages in the garbage cans of her neighbors to look for redeemable cans and bottles. She is very suspicious of her neighbors. She was convinced that her neighbors were plotting against her life for a brief time after she was mugged and thrown onto the pavement by a teenager, but now thinks that this is not the case. She believes in the "power of crystals to protect me" and has them strewn haphazardly through- out her house. Which of the following is the most likely diagnosis? a. Autism b. Schizophrenia, paranoid type c. Schizotypal personality disorder d. Avoidant personality disorder e. Schizoid personality disorder

The answer is c. (Jacobson, pp 61, 187.) Schizotypal personality disorder, a cluster A disorder, is characterized by acute discomfort in close relationships, cognitive and perceptual distortions, and eccentric behavior beginning in early adulthood and present in a variety of contexts. Individ- uals with schizoid personality disorder do not present with the magical thinking, oddity, unusual perceptions, and odd appearance typical of schizotypal individuals. In schizophrenia, psychotic symptoms are much more prolonged and severe. Avoidant individuals avoid social interaction out of shyness and fear of rejection and not out of disinterest or suspi- ciousness. In autism, social interactions are more severely impaired and stereotyped behaviors are usually present.

368. A 52-year-old man comes to the psychiatrist with complaints of problems sleeping. He has problems falling asleep, tossing and turning for several hours before finally getting to sleep. The next day the patient is tired, and this has caused him some problems at work. The patient denies signs or symptoms of major depression. Which of the following is the best sleep hygiene recommendation to help this patient sleep? a. Eat a larger meal near bedtime b. Take daytime naps when possible c. Get up at the same time every day d. Watch television in bed until sleepy e. Begin a graded program of exercise in the early evening

The answer is c. (Kaplan, p 758.) All of the options listed are the opposite of what one would recommend as a sleep hygiene measure, except for the recommendation to get up at the same time every day. Other sleep hygiene methods which can be recommended include: don't take central nervous system stimulants (coffee, nicotine, alcohol) before bed- time, avoid evening television--instead read or listen to the radio, try very hot 20-minute baths before bedtime, and practice evening relaxation rou- tines such as progressive muscle relaxation.

279. A 29-year-old woman comes to a cognitive therapist with a 6-month history of sudden feelings that she is going to die. The patient reports that during these episodes her pulse races, she feels short of breath, and she gets chest pain. She notes that she feels like she is going to die on the spot and therefore has begun to restrict her movement outside the house so that she can remain near a phone in case she needs to call an ambulance. Which of the following treatment interventions should the therapist employ first to begin to help this patient with her problem? a. Taking the patient to a crowded place and preventing her escape until her anx- iety has peaked b. Teaching the patient to hyperventilate as soon as she starts feeling anxious c. Educating the patient about the harmless nature of the physical symptoms experienced during a panic attack d. Taking the patient through a series of imaginary exposures in the therapist's office e. Replying empathically to the patient about the suffering that must be endured with panic attacks

The answer is c. (Kaplan, pp 596.) The cognitive treatment of panic disorder focuses on the patient's tendency to make catastrophic inter- pretations about body sensations or states of mind. This approach includes a careful exploration of the patient's bodily symptoms before and during the panic attack and of the automatic thoughts that accompany them, in addition to an educational component focusing on the fact that, although terrifying, panic attack symptoms are not fatal. Other, more realistic inter- pretations of symptoms are discussed, and the patient is encouraged to come up with less catastrophic scenarios. ("Even if I have a panic attack in a store, the world does not end.") Exposure techniques are part of behav- ioral therapy. Empathy with the patient's suffering is a necessary element of all doctor--patient interactions, but in this case it does not represent a spe- cific therapeutic technique.

321. A 23-year-old woman arrives at the emergency room complaining that, out of the blue, she had been seized by an overwhelming fear, associ- ated with shortness of breath and a pounding heart. These symptoms lasted for approximately 20 minutes, and while she was experiencing them, she feared that she was dying or going crazy. The patient has had four similar episodes during the past month, and she has been worrying that they will continue to recur. Which of the following is the most likely diagnosis? a. Acute psychotic episode b. Hypochondriasis c. Panic disorder d. Generalized anxiety disorder e. Posttraumatic stress disorder

The answer is c. (Jacobson, p 5.) This patient displays typical symp- toms of recurrent panic attacks. Panic attacks can occur under a wide vari- ety of psychiatric and medical conditions. The patient is diagnosed with panic disorder when there are recurrent episodes of panic and there is at least 1 month of persistent concern, worry, or behavioral change associated with the attacks. The attacks are not because of the direct effect of medical illness, medications, or substance abuse and are not better accounted for by another psychiatric disorder. While anxiety can be intense in generalized anxiety disorder, major depression, acute psychosis, and hypochondriasis, it does not have the typical presentation (ie, a discrete episode or panic attack) described in this question.

299. A 22-year-old college student calls his psychiatrist because for the past week, after cramming hard for finals, his thoughts have been racing and he is irritable. The psychiatrist notes that the patient's speech is pres- sured as well. The patient has been stable for the past 6 months on 500 mg of valproate twice a day. Which of the following is the most appropriate first step in the management of this patient's symptoms? a. Hospitalize the patient b. Increase the valproate by 500 mg/day c. Prescribe clonazepam 1 mg qhs d. Start haloperidol 5 mg qd e. Tell the patient to begin psychotherapy one time per week

The answer is c. (Kaplan, p 127.) Sleep deprivation has an antide- pressant effect in depressed patients and may trigger a manic episode in bipolar patients. The patient is not ill enough to require hospitalization. The use of a long-acting benzodiazepine will allow the patient to return to a normal sleep pattern and generally will abort the manic episode.

352. A 32-year-old man is diagnosed with a major depression. He and his psychiatrist discuss starting an antidepressant. The patient is concerned about the chance for impairment of his ability to get an erection on these kinds of medications. Which of the following medications should the patient be started on to treat his depression but avoid these symptoms? a. Imipramine b. Phenelzine c. Fluoxetine d. Desipramine e. Clomipramine

The answer is c. (Kaplan, p 696.) All of the medications listed, except fluoxetine, have been shown to impair the achievement of erections. Fluoxetine, like the other SSRIs, may cause retarded ejaculation. This drug may also lower the sex drive or cause difficulty reaching orgasm in both sexes, probably secondary to the rise in serotonin levels that occur while taking it.

303. A 27-year-old woman has been feeling blue for the past 2 weeks. She has little energy and has trouble concentrating. She states that 6 weeks ago she had been feeling very good, with lots of energy and no need for sleep. She says that this pattern has been occurring for at least the past 3 years, though the episodes have never been so severe that she couldn't work. Which of the following is the most likely diagnosis? a. Borderline personality disorder b. Seasonal affective disorder c. Cyclothymic disorder d. Major depression, recurrent e. Bipolar disorder, depressed

The answer is c. (Kaplan, pp 566-567.) Cyclothymic disorder is characterized by recurrent periods of mild depression alternating with periods of hypomania. This pattern must be present for at least 2 years (1 year for children and adolescents) before the diagnosis can be made. During these 2 years, the symptom-free intervals should not be longer than 2 months. Cyclothymic disorder usually starts during adolescence or early adulthood and tends to have a chronic course. The marked shifts in mood of cyclothymic disorder can be confused with the affective instability of borderline personality disorder or may suggest a substance abuse problem.

273. A 45-year-old man, with no prior psychiatric history, comes to see a therapist. He states that for the past 6 weeks he has been unable to sleep well because he is "stuck." He notes that he has been offered a job that is a large step up from his current one and that this is making him very ner- vous. While he has never seen a therapist before, is very successful at work, and has a good relationship with his wife, he reports that he gets very anx- ious when thinking about this job. He comments that his father was not so successful in the job market, and perhaps this is why the patient cannot move forward. The therapist agrees with the patient's assessment. What kind of therapy is most likely to be used with this patient? a. Psychoanalysis b. Cognitive therapy c. Short-term dynamic psychotherapy d. Supportive psychotherapy e. Experiential-humanistic therapy

The answer is c. (Kaplan, pp 931-933.) This patient, with no psy- chiatric history and otherwise functioning well, is seeing the therapist because he is having a very circumscribed problem with a life event. Patients like these, insightful and otherwise high functioning, generally do better with a short-term dynamic psychotherapy. There is no reason to put the patient through the expense or time of psychoanalysis, and supportive psychotherapy will not help him to get at the root of the problem.

305. A 42-year-old woman sees her physician because she has been depressed for the past 4 months. She also notes that she has gained 20 lb without trying to. She notes that she does not take pleasure in the activities that she once enjoyed and seems fatigued most of the time. These symp- toms have caused the patient to withdraw from many of the social func- tions that she once enjoyed. The physician diagnoses the patient with hypothyroidism and starts her on thyroid supplementation. Six weeks later, the patient's thyroid hormone levels have normalized, but she still reports feeling depressed. Which of the following is the most appropriate next step in the management of this patient? a. Recommend that the patient begin psychotherapy b. Increase the patient's thyroid supplementation c. Start the patient on an antidepressant medication d. Tell the patient that she should wait another 6 weeks, during which time her mood will improve e. Take a substance abuse history from the patient

The answer is c. (Moore and Jefferson, pp 293-295.) This patient is likely suffering from a mood disorder secondary to a general medical con- dition, characterized by a depressed mood and loss of interest in activities she usually enjoyed. It is interesting to note that many of these mood dis- orders, especially those caused by an endocrine disorder, persist even after the underlying medical condition has been treated. In that case (and in the case described in this question), the physician should begin the patient on antidepressant medication.

316. A 32-year-old man is being treated for a severe major depression. Which of the following symptoms, if present, is one of the most accurate indicators of long-term suicidal risk? a. Revenge fantasies b. Presence of rage in the patient c. Hopelessness d. Presence of guilt e. The patient has a need for punishment

The answer is c. (Kaplan, p 900.) A study by Beck showed that hopelessness is one of the most accurate indicators of long-term suicidal risk. No one specific psychodynamic or personality structure is associated with higher risks of suicide. Revenge fantasies, overwhelming affects such as guilt or rage, or the patient's unconscious wish for punishment can all precipitate a suicide attempt.

251. In the vignette above, the patient's verbalization that "when people see me they think I'm a loser" is most often used directly in the context of which kind of therapy? a. Psychodynamic psychotherapy b. Family therapy c. Cognitive therapy d. Behavioral therapy e. Supportive psychotherapy

The answer is c. (Kaplan, p 959.) Once cognitive distortions are recognized and characterized, cognitive therapy is used to unravel them by testing them, identifying their maladaptive underlying assumptions, and testing the validity of those assumptions as well.

295. A 30-year-old man comes to the psychiatrist for the evaluation of a depressed mood. He states that at least since his mid-20s he has felt depressed. He notes poor self-esteem and low energy, and feels hopeless about his situation, though he denies suicidal ideation. He states he does not use drugs or alcohol, and has no medical problems. His last physical examination by his physician 1 month ago was entirely normal. Which of the following treatment options should be tried first? a. ECT b. Hospitalization c. Psychoanalysis d. Venlafaxine e. Amoxapine

The answer is d. (Kaplan, pp 563-566.) This patient has a dys- thymic disorder. While many clinicians do not believe that these disorders should be treated pharmacologically, there are a number of studies that show positive responses to antidepressants with these patients. Venlafaxine and bupropion are generally believed to be the treatments of choice for dysthymic disorder, though there is a subgroup of patients that will respond to the MAOIs as well.

358. A demanding 25-year-old woman begins psychotherapy stating that she is both desperate and bored. She reports that for the past 5 or 6 years she has experienced periodic anxiety and depression and has made several suicidal gestures. She also reports a variety of impulsive and self-defeating behaviors and sexual promiscuity. She wonders if she might be a lesbian, though most of her sexual experiences have been with men. She has abruptly terminated two previous attempts at psychotherapy. In both cases she was enraged at the therapist because he was unwilling to prescribe anx- iolytic medications. Which of the following is the most likely diagnosis? a. Dysthymia b. Histrionic personality disorder c. Antisocial personality disorder d. Borderline personality disorder e. Impulse control disorder not otherwise specified

The answer is d. (Ebert, pp 523-526.) The patient's history and pre- senting symptoms are classic for the diagnosis of borderline personality disorder. Patients with borderline personalities present with a history of a pervasive instability of mood, relationships, and self-image beginning by early adulthood. Their behavior is often impulsive and self-damaging, their sexuality is chaotic, sexual orientation may be uncertain, and anger is intense and often acted out. Recurrent suicidal gestures are common.

331. Which of the following is the treatment of choice for the patient described in the previous vignette? a. Imipramine b. Clonazepam c. Propanolol d. Exposure therapy e. Psychoanalysis

The answer is d. (Jacobson, p 217.) No medication has proven to be effective in treating specific phobias. The treatment of choice for specific phobias is exposure, in vivo or using techniques of guided imagery, pairing relaxation exercises with exposure to the feared stimulus. The patient can be exposed to the feared stimulus gradually or can be asked to immediately confront the most anxiety-provoking situation (flooding).

330. A 34-year-old secretary climbs 12 flights of stairs every day to reach her office because she is terrified by the thought of being trapped in the ele- vator. She has never had any traumatic event occur in an elevator; nonethe- less, she has been terrified of them since childhood. Which of the following is the most likely diagnosis? a. Social phobia b. Performance anxiety c. Generalized anxiety disorder d. Specific phobia e. Agoraphobia

The answer is d. (Jacobson, p 80.) Specific phobias are character- ized by an unreasonable or excessive fear of an object, an animal, or a situ- ation (flying, being trapped in close spaces, heights, blood, spiders, etc.). Since the exposure to the feared situation, animal, or object causes an immediate surge of anxiety, patients carefully avoid the phobic stimuli. The diagnosis of specific phobia requires the presence of reduced functioning and interference with social activities and relationships because of the avoidant behavior, anticipatory anxiety, and distress caused by the expo- sure to the feared stimulus. In social phobias and performance anxiety, patients fear social interactions (in general or limited to specific situations) and public performance (public speaking, acting, playing an instrument), respectively. In generalized anxiety disorder, the anxiety is more chronic and less intense than in a phobic disorder and is not limited to a specific situation or item. Agoraphobic patients fear places where escape may be difficult or help may not be available in case the patient has a panic attack. Agoraphobic patients are often prisoners in their own homes and depend on a companion when they need to go out.

306. A 54-year-old man sees a physician complaining of a depressed mood and inability to sleep for the past 3 weeks. He tells the physician that in the past when he has had similar episodes, he was placed on a monoamine oxidase inhibitor, which proved effective. The physician diag- noses the patient with major depression and agrees to use an MAOI. Which of the following foods must be completely avoided by this patient while on this medication? a. Licorice b. Coffee c. Chocolate d. Cheddar cheese e. Soy sauce

The answer is d. (Jacobson, pp 121-122.) There is a long list of foods that should be avoided while a patient is taking a monoamine oxidase inhibitor, but only a few that must be completely avoided--these include aged cheese, red wine, and cured meats. Others, including anchovies, beets, caviar, chocolate, coffee, colas, curry powder, figs, licorice, mushrooms, rhubarb, snails, soy sauce, and Worcestershire sauce, can be consumed in small quantities because although they contain small amounts of tyramine, they have only rarely been associated with hypertensive episodes.

302. A 38-year-old woman with bipolar disorder has been stable on lithium for the past 2 years. She comes to her psychiatrist's office in tears after a 2-week history of a depressed mood, poor concentration, loss of appetite, and passive suicidal ideation. Which of the following is the most appropriate next step in the management of this patient? a. Start the patient on a second mood stabilizer b. Start the patient on a long-acting benzodiazepine c. Stop the lithium and start an antidepressant d. Start an antidepressant and continue the lithium e. Stop the lithium and start an antipsychotic

The answer is d. (Kaplan, p 561.) Since lithium and other mood sta- bilizers are more effective in the prevention of manic episodes than in the prevention of depression, antidepressants are used as an adjunctive treat- ment when depressive episodes develop during maintenance with a mood stabilizer. Since the incidence of antidepressant-induced mania is high (up to 30%), and since antidepressant treatment may cause rapid cycling, the antidepressant should be tapered and discontinued as soon as the depres- sive symptoms remit. Among the antidepressants in common use, bupro- pion is considered to carry a slightly lower risk of triggering mania.

369. A 21-year-old man comes to the physician because of excessive sleepiness. He states that for the past 4 months he becomes so sleepy that he must sleep, even when he is in the middle of an important meeting. These episodes occur daily and the patient must sleep for 10 to 20 minutes at each episode. The patient also says that on several occasions he has had a sudden loss of muscle tone during which his knees become weak and he drops to the floor. He remains conscious during these episodes. He denies any substance abuse or medical problems. Which of the following is the most appropriate treatment to be started? a. Benzphetamine b. Valproic acid c. Lithium d. Modafinil e. Nasal continuous positive airway pressure

The answer is d. (Kaplan, p 760.) Modafinil (Provigil) has been approved by the FDA to reduce the number of sleep attacks and to lessen cataplexy (the sudden loss of muscle tone which is causing this patient's knees to buckle). Modafinil, like other stimulants, increases the release of monoamines, but also elevates hypothalamic histamine levels. Patients can develop tolerance to this drug and should be monitored closely while on it. It does lack some of the adverse side effects of psychostimulants, which were previously used to treat this disorder.

355. Every 4 or 5 weeks, a usually well-functioning and mild-mannered 35-year-old woman experiences a few days of irritability, tearfulness, and unexplained sadness. During these days, she also feels fatigued and bloated and eats large quantities of sweets. Which of the following is the most likely diagnosis? a. Cyclothymia b. Borderline personality disorder c. Dissociative identity disorder d. Premenstrual dysphoric disorder e. Minor depressive disorder

The answer is d. (Kaplan, p 867.) The physical and emotional symptoms of premenstrual dysphoric disorder (PMDD) are restricted to the late luteal phase of the menstrual cycle and resolve 1 or 2 days after the onset of menstruation. Although most women of childbearing age experi- ence some symptoms of PMDD during some of their menstrual cycles, only 5% to 9% meet the criteria for the diagnosis.

311. A 10-year-old boy is brought to the psychiatrist by his mother. She states that for the past 2 months he has been increasingly irritable, withdrawn, and apathetic. He has been refusing to do his homework, and his grades have dropped. Which of the following is the best next step in management? a. The child should be hospitalized. b. The child should be started in supportive psychotherapy. c. The mother should be warned that the child will likely turn out to be bipolar (67% chance). d. The child should receive an antidepressant medication. e. The child should receive lithium and an antidepressant.

The answer is d. (Kaplan, pp 1260-1262.) Major depression is not a rare occurrence in children. Prevalence rates are 0.3% in preschoolers, 0.4% to 3% in school-age children, and 0.4% to 4.6% in adolescents. Mak- ing a correct diagnosis is complicated by the fact that the presentation of juvenile depression often differs from the adult presentation. Depressed preschoolers tend to be irritable, aggressive, withdrawn, or clingy instead of sad. In school-age children, the main manifestation of depression may be a significant loss of interest in friends and school. By adolescence, pre- senting symptoms of depression become more similar to those of adults. Psychotic symptoms are common in depressed children, most commonly one voice that makes depreciative comments and mood-congruent delu- sional ideations. Up to one-third of children diagnosed with major depression receive a diagnosis of bipolar disorder later in life. This evolution is more likely when the depressive episode has an abrupt onset and is accompanied by psychotic symptoms. Childhood depression can be treated pharmaco- logically, but child response to medication differs from adult response. SSRIs have been proven effective in preschoolers and school-age children; tricyclic antidepressants (TCAs) have not. There are insufficient data about the efficacy of newer antidepressants such as nefazodone, venlafaxine, bupropion, and mirtazapine. The response of older adolescents to antide- pressants is equivalent to the adult response.

271. A 27-year-old man comes to the physician with the chief complaint of premature ejaculation. He has been married for 4 months but has been unable to consummate the marriage because of his sexual problem. No organic cause for his premature ejaculation was found on work-up. Which of the following treatments will be most helpful for the man's premature ejaculation? a. Exploration of the husband's relationship with his domineering mother b. Discussion of the wife's unexpressed masochistic fantasies c. Interpretation of the husband's dreams d. Squeeze technique and stop-and-start technique e. Instructing the husband to masturbate several times a day with the goal to reach an orgasm as fast as possible

The answer is d. (Kaplan, pp 701-705.) Treatment of sexual dys- functions relies on specific exercises, called sensate focus exercises, aimed at decreasing anxiety, to teach the couple to give and take pleasure without the pressure of performance, and to increase communication between part- ners. Furthermore, specific problems are addressed with special tech- niques. The squeeze technique, used to treat premature ejaculation, aims to raise the threshold of penile excitability by firmly squeezing the coronal ridge of the penis, so as to abruptly decrease the level of excitation, at the earliest sensation of impending orgasm. In the start-and-stop technique, stimulation is repeatedly stopped for a few seconds as soon as orgasm is impending and resumed when the level of excitability decreases.

266. A 48-year-old woman comes to the psychiatrist because she has an overwhelming fear of spiders. She has had this fear her entire life, but it has increased now secondary to living in a wooded area where there are greater numbers of them. She wishes to get rid of this phobia. Which of the fol- lowing actions should the psychiatrist take next? a. Prescribe alprazolam b. Prescribe bupropion c. Prescribe propranolol d. Have the patient create a hierarchical list of feared situations involving spiders e. Engage the patient in psychodynamic psychotherapy to get at the root of her arachnophobia

The answer is d. (Kaplan, p 953.) All behavioral treatments for phobias have in common exposure to the feared stimulus. Desensitization is based on the concept that when the feared stimulus is presented paired with a behavior that induces a state incompatible with anxiety (eg, deep muscle relaxation), the phobic stimulus loses its power to create anxiety (counterconditioning). This pairing of feared stimulus with a state incom- patible with anxiety is called reciprocal inhibition. For desensitization to work, the anxiety elicited by the exposure has to be low. Treatment starts with exposure to stimuli that produce minimal anxiety and proceeds to stimuli with higher anxiety potential. Operant conditioning refers to the concept that behavior can be modified by changing the antecedents or the consequences of the behavior (contingency management). Flooding is another exposure-based treatment for phobia, based on extinction rather than counterconditioning. Reframing is an intervention used in family therapy and refers to giving a more acceptable meaning to a problematic behavior or situation.

274. A high school teacher is respected and loved by both his students and his colleagues because he can easily defuse tense moments with an appropriate light remark and he always seems to be able to find something funny in any situation. Which of the following defense mechanisms is this man using? a. Displacement b. Denial c. Reaction formation d. Humor e. Suppression

The answer is d. (Kaplan, pp 202-203.) Individuals who use humor as a defense mechanism are able to make use of comedy to express feelings and thoughts with potentially disturbing content without experiencing subjective discomfort and without producing an unpleasant effect on others. Humor is a mature defense. Suppression is consciously or semi- consciously postponing attention to a conscious impulse or conflict. In suppression, discomfort is acknowledged but minimized.

253. A 26-year-old man comes to the psychiatrist because he becomes extremely anxious in social situations. He is unable to talk to anyone and experiences sweaty palms and a rapid heartbeat. Which of the following treatment options will provide the most effective and longest lasting results for this patient? a. Psychodynamic psychotherapy b. Treatment with an antidepressant c. Psychoanalysis d. Cognitive-behavioral therapy e. Treatment with an anxiolytic

The answer is d. (Kaplan, pp 602-604.) Although medication can be effective in the treatment of social phobia, cognitive-behavioral therapy produces results that are just as effective, and probably longer lasting. There are no data to support the use of psychodynamic psychotherapy or psychoanalysis for the treatment of social phobia.

298. How long after a stroke is a patient at a higher risk for developing a depressive disorder? a. 2 weeks b. 2 months c. 6 months d. 1 year e. 2 years

The answer is e. (Kaplan, p 1086.) Studies of the course and prog- nosis of poststroke depression indicate that the high-risk period can last up to 2 years. The presence of depression is associated with an eightfold increase in mortality risk.

300. A 24-year-old woman, 5 days after delivery of a normal, full-term infant, is brought to the obstetrician because she is so tearful. She states that her mood is quite labile, often changing within minutes. She has trou- ble sleeping, both falling asleep and awakening early. She notes anhedonia, stating she doesn't enjoy "much of anything" right now. Which of this patient's symptoms point preferentially to a postpartum depression? a. Time--that is, 5 days post delivery b. Tearfulness c. Labile mood d. Insomnia e. Anhedonia

The answer is e. (Kaplan, p 865.) All of the symptoms that this patient is experiencing are congruent with the postpartum blues. These symptoms may last several days, and are thought to be secondary to the combination of large hormonal shifts and the awareness of increased responsibility for a new human being. Anhedonia is not seen in postpar- tum blues, but is common in postpartum depressions.

291. A 25-year-old man comes to the psychiatrist with a chief complaint of depressed mood for 1 month. His mother, to whom he was very close, died 1 month ago, and since that time he has felt sad and been very tearful. He has difficulty concentrating, has lost 3 lb, and is not sleeping soundly through the night. Which of the following is the most likely diagnosis? a. Major depression b. Dysthymia c. Posttraumatic stress disorder d. Adjustment disorder e. Uncomplicated bereavement

The answer is e. (Kaplan, pp 64-68.) The loss of a loved one is often accompanied by symptoms reminiscent of major depression, such as sad- ness, weepiness, insomnia, reduced appetite, and weight loss. When these symptoms do not persist beyond 2 months after the loss, they are consid- ered a normal manifestation of bereavement. A diagnosis of major depres- sion in these circumstances requires the presence of marked functional impairment, morbid preoccupations with unrealistic guilt or worthless- ness, suicidal ideation, marked psychomotor retardation, and psychotic symptoms, on top of the symptoms listed in the first sentence above. A diagnosis of adjustment disorder with depressed mood would not nor- mally be given to someone when the "adjustment" is to the recent death of a loved one--instead, bereavement is the diagnosis given (complicated or uncomplicated).

400. A 37-year-old woman is admitted to an inpatient treatment program for withdrawal from heroin. Eighteen hours after her last injection of heroin, she becomes hypertensive, irritable, and restless. She also has nau- sea, vomiting, and diarrhea. Which medication would be best to treat some of the symptoms of opioid withdrawal? a. Chlordiazepoxide b. Haloperidol c. Paroxetine d. Phenobarbital e. Clonidine

The answer is e. (Moore and Jefferson, p 80.) Clonidine, an alpha- 2-adrenergic receptor agonist, is used to suppress some of the symptoms of mild opioid withdrawal. Clonidine is given orally, starting with doses of 0.1 to 0.3 mg three or four times a day. In outpatient settings, a daily dosage above 1 mg is not recommended because of the risk of severe hypotension. Clonidine is more effective on symptoms of autonomic instability, but is less effective than methadone in suppressing muscle aches, cravings, and insomnia. Clonidine is particularly useful in the detoxification of patients maintained on methadone.

255. A patient perceives his analyst as wise, caring, and helpful. During his session, he talks at length about his warm feelings toward the therapist. Which of the following is the most appropriate next step the analyst should take? a. Tell the patient that he does not really feel this way--he is experiencing transference. b. Tell the patient that his positive feelings cannot be reciprocated. c. Tell the patient that these feelings are not helpful in the service of the therapy. d. Tell the patient that underneath the positive feelings are undoubtedly negative ones. e. Ask the patient to explore related feelings he has about the topic.

The answer is e. (Kaplan, p 207.) Freud believed that unobjection- able positive transference, defined as the patient's perception of the therapist as caring and helpful, is always helpful to the therapeutic process and should not be analyzed. Critics of Freud's theories pointed out that some- times an overt positive transference may conceal less flattering feelings. Fur- thermore, patients' positive perceptions of the therapist are not necessarily all projections from past experiences, since the therapist's personality and behavior have powerful effects on the form and content of the transference.

272. A 49-year-old man comes to the doctor with high blood pressure and anxiety. Preferring to try something other than medication at first, the patient agrees to try another approach. He is attached to an apparatus that measures skin temperature and emits a tone proportional to the tempera- ture. Which of the following techniques is being used with this patient? a. Hypnosis b. Progressive muscle relaxation c. Autogenic techniques d. Placebo e. Biofeedback

The answer is e. (Kaplan, pp 949-953.) Biofeedback refers to a ther- apeutic process in which information about the individual's physiological functions, such as blood pressure and heart rate, are monitored electroni- cally and fed back to the individual by means of lights, sounds, or elec- tronic gauges. Biofeedback allows individuals to control a variety of body responses and in turn to modulate pain and the physiological component of unpleasant emotions such as anxiety.

Questions 360 to 363 Match each patient's behavior with the most likely personality disorder. Each lettered option may be used once, more than once, or not at all. a. Paranoid b. Schizotypal c. Schizoid d. Narcissistic e. Borderline f. Histrionic g. Antisocial h. Obsessive-compulsive i. Dependent j. Avoidant 360. A 28-year-old woman begins seeing a psychiatrist because, she says, "I am just so very lonely." Her speech is excessively impressionistic and lacks specific detail. She flirts constantly with the physician and is "hurt" when the therapist does not notice her new clothes or hairstyle. 361. A 42-year-old man comes to the psychiatrist at the insistence of his boss because he constantly misses important deadlines. The man states that everyone at work is lazy and that no one lives up to his own standards for perfection. He is angry when the physician starts the interview 3 minutes later than the appointed time. He notes that he is always fighting with his wife because he is a "pack rat" and is unable to throw anything out. During the interview, he appears very rigid and stubborn. 362. A 34-year-old woman comes to the psychiatrist on the advice of her mother, because the patient still lives at home and will not make any deci- sions without her mother's reassurance. The patient's mother accompanies the patient to the appointment. She states that the patient becomes anxious when her mother must leave the home because the patient is terrified that her mother will die and the patient will have to take care of herself, some- thing she feels incapable of doing. 363. A 25-year-old high school dropout has been arrested more than 12 times for various assault, fraud, and attempted murder charges. He has been in many physical fights, usually after he got caught cheating at cards. On examination, he seems relaxed and even cocky, and he shows no remorse for his actions.

The answers are 360-f, 361-h, 362-i, 363-g. Histrionic personality disorder is characterized by a chronic pattern of excessive emotionality and attention seeking. Patients with this disorder like to be the center of attention, and interaction with others is often inappropriately seductive. These patients may also use their physical appearance to draw attention to themselves. Speech is dramatic but super- ficial, and details are lacking. Patients are easily suggestible and influenced by others. Patients with obsessive-compulsive personality disorder are pre- occupied with orderliness, perfection, and control. They are often so pre- occupied with details, lists, and order that they lose sight of the forest for focusing on the trees. Their concern with perfectionism makes it difficult for them to complete projects in a timely manner. They are often perceived as rigid and stubborn. Patients with dependent personality disorder have a chronic and excessive need to be taken care of. This leads to submissive, clinging behavior in a desperate attempt to avoid being separated from the caretaker. These patients have difficulty making everyday decisions on their own and often need considerable reassurance before being able to do so. They are also unrealistically preoccupied with fears of being left alone to take care of themselves. Individuals with antisocial personality disorder display a chronic pattern of disregard for the rights of others, and they often violate them. They also are frequently irritable and aggressive, engag- ing in repeated physical assaults. They do not show remorse for their activ- ities. This diagnosis is not made if the episodes of antisocial behavior occur exclusively during the course of schizophrenia or a manic episode.

Questions 376 to 377 376. An off-Broadway actor consistently bores his friends and acquain- tances by talking incessantly about his exceptional talent and his success on the stage. He does not seem to realize that other people do not share his high opinion of his acting talent and are not interested in his monologues. When a director criticizes the way he delivers his lines during rehearsal, the actor goes into a rage and accuses the director of trying to jeopardize his career out of jealousy. Which personality disorder represents the most likely diagnosis? a. Histrionic b. Narcissistic c. Borderline d. Paranoid e. Antisocial 377. The patient in the vignette above seeks out a psychiatrist because, he says, "It is depressing when no one understands your talent." Which of the following treatments would be most appropriate? a. Medication with an SSRI b. Medication with a tricyclic antidepressant c. Group psychotherapy with patients from a wide range of other diagnoses d. Psychoanalysis e. Psychodynamic psychotherapy

The answers are 376-b, 377-e. (Kaplan, p 803.) The essential feature of narcissistic personality disorder is a pervasive pattern of grandiosity, need for admiration, and lack of empathy that begins by early adulthood. Individuals with this disorder overestimate their abilities, inflate their accomplishments, and expect others to share the unrealistic opinion they have of themselves. They believe they are special and unique and attribute special qualities to those with whom they associate. When they do not receive the admiration they think they deserve, people with narcissistic personality react with anger and devaluation. The prevalence of the disorder is estimated at less than 1% of the general population, and 50% to 75% of those diagnosed with narcissistic personality are males. In contrast with their outward appearance, individuals with this disorder have a very vulnerable sense of self. Criticism leaves them feeling degraded and hollow. Narcissistic traits are common in adolescence, but most individuals do not progress to develop narcissistic personality disorder. Treatment of narcissistic personality disorder is extremely difficult and requires a tactful therapist who can make confrontations, but do it gently. Forming an alliance with these patients can be very difficult. Medications do not work for this disorder. Psychoanalysis would be too intense for a patient with this disorder, and the abstinent stance would quickly drive the patient from therapy. Likewise, group therapy with a heterogeneous group would likely enrage a narcissist, who would be unable to take criticism from the other group members. Sometimes homogeneous groups of patients (a group with all narcissists, for example) might be able to work together therapeutically because it would help them understand their own maladaptive patterns as they watch others' behaviors.

250. A 37-year-old man comes to the psychiatrist for treatment of a depressed mood. The patient has anhedonia, anergia, decreased concentra- tion, obsessive ruminations of guilt, insomnia, and a 5-lb weight loss over the past 2 weeks. He avoids going out in public secondary to the belief that people won't like him. In the psychiatrist's office, he was helped to verbal- ize the fact that whenever he met new people, his immediate reaction was to believe that these people could see he was "a loser." This last is a verbal- ization of which kind of behavior, often seen in patients with this diagnosis? a. Automatic thought b. Delusion c. Obsession d. Avoidance e. Modeling

answer is a. (Kaplan, p 959.) Automatic thoughts (cognitive distortions) are thoughts that come between the time an event occurs externally and the person having the thought has an emotional reaction to the external event. For example, the belief that "I am so ugly" is an auto- matic thought that may occur in between a person complimenting another on their new dress (the external event) and the person in the dress exclaim- ing, "You must be crazy!" (Prompted by the emotional reaction of dismay.) Every psychopathological disorder has its own particular profile of dis- torted thoughts, which if known, can provide the framework for cognitive work to stop them.

301. A 28-year-old woman sees her physician with the chief complaint of a depressed mood. She also notes that she is sleeping more than usual--up to 14 hours per night--but does not feel rested and that she feels tired and fatigued all the time. She has gained 14 lb in the last month, something that she is very unhappy about, but she says that she seems to have such a crav- ing for sweets that the weight gain seemed inevitable. Which of the follow- ing is the most likely diagnosis? a. Mood disorder secondary to a general medical condition b. Substance-induced mood disorder c. Cyclothymia d. Seasonal affective disorder e. Dysthymic disorder

answer is d. (Moore and Jefferson, pp 136, 140.) Patients with seasonal affective disorder typically present in just the way this patient has. Patients usually exhibit typical signs and symptoms seasonally, most often during the winter, with symptoms remitting in the spring. Hypersomnia and hyperphagia (atypical signs of a depression) are classically seen with this disorder. Light therapy and serotonergic agents (typically SSRIs) are the treatments of choice for this disorder.

Questions 317 to 320 Match each patient's symptoms with the most appropriate diagnosis. Each lettered option may be used once, more than once, or not at all. a. Atypical depression b. Double depression c. Cyclothymic disorder d. Melancholic depression e. Schizoaffective disorder f. Seasonal affective disorder 317. An elderly man has been profoundly depressed for several weeks. He cries easily and is intensely preoccupied with trivial episodes from his past, which he considers unforgivable sins. This patient awakens every morning at 3 AM and cannot go back to sleep. Anything his family has tried to cheer him up has failed. He has completely lost his appetite and appears gaunt and emaciated. 318. A young woman, who has felt mildly unhappy and dissatisfied with herself for most of her life, has been severely depressed, irritable, and anhe- donic for 3 weeks. 319. For the past 6 weeks, a middle-aged woman's mood has been mostly depressed, but she cheers up briefly when her grandchildren visit or in coincidence with other pleasant events. She is consistently less depressed in the morning than at night. When her children fail to call on the phone to inquire about her health, her mood deteriorates even more. She sleeps 14 hours every night and has gained 24 lb. 320. Since he moved to Maine from his native Florida 3 years earlier, a college student has had great difficulty preparing for the winter-term courses. He starts craving sweets and feeling sluggish, fatigued, and irrita- ble in late October. These symptoms worsen gradually during the follow- ing months, and by February he has consistently gained several pounds. His mood and energy level start improving in March, and by May he is back to baseline.

answers are 317-d, 318-b, 319-a, 320-f Melancholic depression, a variant of major depressive disorder, is characterized by loss of pleasure in all activities (anhedonia), lack of reactivity (nothing can make the patient feel better), intense guilt, signifi- cant weight loss, early morning awakening, and marked psychomotor retardation. TCAs have been considered to be more effective than other antidepressants in the treatment of melancholic depression. Double depression is diagnosed when a major depressive episode develops in a patient with dysthymic disorder. Between 68% and 90% of patients with dysthymic disorder experience at least one episode of major depression during their lives. Compared with patients who are euthymic between depressive episodes, dysthymic patients with superimposed major depression experience a higher risk for suicide, more severe depressive symptoms, more psychosocial impairment, and more treatment resistance. Atypical depression, another variant of major depressive disorder, is characterized by mood reactivity (pleasurable events may temporarily improve the mood), self-pity, excessive sensitivity to rejection, reversed diurnal mood fluctuations (patients feel better in the morning), and reversed vegetative symptoms (increased appetite and increased sleep). Approximately 15% of patients with depression have atypical features. MAOIs are considered to be more effective than other classes of antide- pressants in atypical depression.

Questions 323 to 327 Match the following classical presentations with its diagnosis. Each lettered option may be used once, more than once, or not at all. a. Somatization disorder b. Conversion disorder c. Hypochondriasis d. Body dysmorphic disorder e. Pain disorder 323. A 20-year-old woman comes to her primary care doctor with multi- ple symptoms which are present across several organ systems. She has seen five doctors in the past 3 months, and has had six surgeries since the age of 18. 324. A 24-year-old woman from a rural and low socioeconomic back- ground with a fifth-grade education develops sudden left-arm paralysis that is not compatible with known neural patterns. 325. A 49-year-old man calls his physician repeatedly demanding a workup for his severe back pain. The physical examination is within nor- mal limits and a CT of his back is also normal, but this is only temporarily reassuring to the patient. 326. A 17-year-old girl presents to a physician complaining that her face is "out of proportion" and that she looks like "Mr Hyde--like a monster." On examination, the girl is a pleasant-looking young woman with no facial deformities of any kind. 327. A 45-year-old woman presents to her physician with a chief com- plaint of a severe headache that is increasing in severity over the past 3 weeks. The patient states that 1 month ago she was in an auto accident and was diagnosed with a concussion. The patient states that the headache has been increasing since then and she is completely unable to work. The MRI of her head is normal.

answers are 323-a, 324-b, 325-c, 326-d, 327-e

390. A 28-year-old woman is seen for postpartum blues by the psychiatrist. She states she is depressed because she "did this to her child." The infant has growth retardation, microphthalmia, short palpebral fissures, midface hypoplasia, a short philtrum, a thin upper lip, and microcephaly. Which is the most likely diagnosis of the mother (besides the postpartum blues)? a. Bipolar disorder b. Major depression c. Hypochondriasis d. Alcohol dependence e. Cocaine dependence

he answer is d. (Kaplan, p 20.) This woman is likely suffering from alcohol dependence, because her child is showing the classic signs of a fetal alcohol syndrome. This syndrome affects approximately one-third of all infants born to women afflicted with alcoholism. Besides the signs this infant has, one can see delayed development, hyperactivity, attention deficits, learn- ing disabilities, intellectual deficits, and seizures in these children.


Kaugnay na mga set ng pag-aaral

ANATOMY KEY CONCEPTS CH 6.1-6.12

View Set

Chapter 12: Current Liabilities and Contingencies

View Set

2 - Introduction to Forwards and futures

View Set

Chapter 19 Multiple Choice- Computational

View Set